0
$\begingroup$

I wonder whether the sequence $s_{n} = \sum_{k = 1}^{n} \sin k$ is bounded.

The answer seems no, but I have no idea how to prove this from the irrationality of $\pi$.

$\endgroup$
1
  • 5
    $\begingroup$ It's not a research level question. You should ask it at math.stack.exchange. $\endgroup$ Jun 9, 2012 at 11:31

1 Answer 1

11
$\begingroup$

In this particular case, use $s_n(\theta)=\sum_{k=0}^n\sin(k\theta)=\Im(\sum_{k=0}^n\exp(ki\theta))=\dots=\frac{\sin(n\theta/2)\sin((n+1)\theta/2)}{\sin (\theta/2)}$

$\endgroup$
1
  • 1
    $\begingroup$ In particular, $s_n$ is bounded. $\endgroup$ Jun 9, 2012 at 11:55

Not the answer you're looking for? Browse other questions tagged or ask your own question.